Premise Validity in GMAT: Help required David Sir

This topic has expert replies
User avatar
Master | Next Rank: 500 Posts
Posts: 377
Joined: Wed Sep 14, 2011 10:45 am
Thanked: 10 times
Followed by:1 members
At times, we find that the argument has a Claim/Prediction functioning as a Premise for Main Conclusion.

Do we always consider those claims and predictions as TRUE?

If so, why? since they are not FACT and can be questioned.
If not, why? since Premise are not questioned in GMAT.

User avatar
Legendary Member
Posts: 2663
Joined: Wed Jan 14, 2015 8:25 am
Location: Boston, MA
Thanked: 1153 times
Followed by:128 members
GMAT Score:770

by DavidG@VeritasPrep » Wed Jun 14, 2017 6:34 am
imskpwr wrote:At times, we find that the argument has a Claim/Prediction functioning as a Premise for Main Conclusion.

Do we always consider those claims and predictions as TRUE?

If so, why? since they are not FACT and can be questioned.
If not, why? since Premise are not questioned in GMAT.
I'm guessing you're referring to this type of scenario:

Premise: Investors believe that the bank is going to fail
Conclusion: Therefore, there will be a run on the bank

Here's how to think about the premise. It is definitely true that investors believe that the bank is going to fail. But it clearly doesn't have to be the case that the investors are right. In other words, even if the bank doesn't fail, the investors still harbored the belief that it would, so the premise would not have been undermined. (But the conclusion, of course, is another story.)
Veritas Prep | GMAT Instructor

Veritas Prep Reviews
Save $100 off any live Veritas Prep GMAT Course

User avatar
Master | Next Rank: 500 Posts
Posts: 377
Joined: Wed Sep 14, 2011 10:45 am
Thanked: 10 times
Followed by:1 members

by imskpwr » Thu Jun 15, 2017 7:29 pm
If I say, a recently aired news that the banks may get govt support raised doubts among investors.

Can I eliminate such considerations in GMAT, without a second thought?

User avatar
Legendary Member
Posts: 2663
Joined: Wed Jan 14, 2015 8:25 am
Location: Boston, MA
Thanked: 1153 times
Followed by:128 members
GMAT Score:770

by DavidG@VeritasPrep » Fri Jun 16, 2017 11:18 am
imskpwr wrote:If I say, a recently aired news that the banks may get govt support raised doubts among investors.

Can I eliminate such considerations in GMAT, without a second thought?
I'm not quite sure what you're asking here. Are you proposing a scenario in which the above appears as part of the original argument and wondering if such a premise would be ironclad?
Veritas Prep | GMAT Instructor

Veritas Prep Reviews
Save $100 off any live Veritas Prep GMAT Course

User avatar
Master | Next Rank: 500 Posts
Posts: 228
Joined: Thu Apr 20, 2017 1:02 am
Location: Global
Thanked: 32 times
Followed by:3 members
GMAT Score:770

by elias.latour.apex » Tue Jun 20, 2017 11:12 am
Let's assume that you are working on a weaken question. We are trying to undermine the conclusion.

Our first step is to identify that conclusion. Let's take a simple argument:

John is tall. Accordingly, he must be good at basketball.

The blue statement is the conclusion. Once we know the conclusion, we can ask the question why? Because John is tall. But our job is to weaken the conclusion.

You seem to be asking whether we could dispute the idea that John is tall. How tall is he? Maybe he's only 5'11. Is that really tall? However, these types of considerations will not be part of a GMAT argument.

The GMAT argument will revolve around the assumption. In this case the assumption is: Tall people are good at basketball. We know that this is the assumption because the conclusion mentions basketball, but the premise only talks about John's height. The assumption will connect the premise to the conclusion.

In order to weaken the argument, we will be trying to attack that assumption. How can we do so?

One way is to simply have a statement that negates the assumption. One example may be "Not all tall people are good at basketball." Another way may be to show that the cause does not always lead to the effect. A statement such as "Carlos is taller than John, but he's terrible at basketball" would suffice.

Here's a real example:

A drug that is highly effective in treating many types of infection can, at present, be obtained only from the bark of the ibora, a tree that is quite rare in the wild. It takes the bark of 5,000 trees to make one kilogram of the drug. It follows, therefore, that continued production of the drug must inevitably lead to the ibora's extinction.

Which of the following, if true, most seriously weakens the argument above?

(A) The drug made from ibora bark is dispensed to doctors from a central authority.
(B) The drug made from ibora bark is expensive to produce.
(C) The leaves of the ibora are used in a number of medical products.
(D) The ibora can be propagated from cuttings and grown under cultivation.
(E) The ibora generally grows in largely inaccessible places.
--------------------------------------------------------------------------
What's the conclusion? It is: Continued production of the drug will lead to the ibora's extinction.
Why? Because A) the drug can only be obtained from the bark of the ibora and b) the ibora is quite rare in the wild.
What's the assumption? That wild ibora trees are the only source of ibora bark. Accordingly, to weaken, we will need to find some other source of ibora bark than wild ibora trees. Answer choice (D), the credited response, does so.

The best answer will never be something like: Ibora trees are actually quite common in the wild or It actually takes the bark from only one ibora tree to make the drug.
Elias Latour
Verbal Specialist @ ApexGMAT
blog.apexgmat.com
+1 (646) 736-7622

User avatar
Master | Next Rank: 500 Posts
Posts: 377
Joined: Wed Sep 14, 2011 10:45 am
Thanked: 10 times
Followed by:1 members

by imskpwr » Mon Jun 26, 2017 7:52 am
DavidG@VeritasPrep wrote: I'm not quite sure what you're asking here. Are you proposing a scenario in which the above appears as part of the original argument and wondering if such a premise would be ironclad?
Premise: Investors believe that the bank is going to fail
Conclusion: Therefore, there will be a run on the bank

Q: We are asked to Weaken the Conclusion.

One of the answer choice is: a recently aired news that the banks may get govt support raised doubts among investors.

Can I straight way eliminate such a choice on the basis that it raises doubts on the Validity of the premise?

User avatar
Master | Next Rank: 500 Posts
Posts: 377
Joined: Wed Sep 14, 2011 10:45 am
Thanked: 10 times
Followed by:1 members

by imskpwr » Mon Jun 26, 2017 8:07 am
elias.latour.apex wrote: Here's a real example:

A drug that is highly effective in treating many types of infection can, at present, be obtained only from the bark of the ibora, a tree that is quite rare in the wild. It takes the bark of 5,000 trees to make one kilogram of the drug. It follows, therefore, that continued production of the drug must inevitably lead to the ibora's extinction.
To weaken,

If an answer choice says that scientists have now found a way to obtain this drug from sources other than the bark of Ibora tree, can I eliminate it directly on the basis that it raises doubts on the validity of premise.

User avatar
Legendary Member
Posts: 2663
Joined: Wed Jan 14, 2015 8:25 am
Location: Boston, MA
Thanked: 1153 times
Followed by:128 members
GMAT Score:770

by DavidG@VeritasPrep » Mon Jun 26, 2017 8:12 am
imskpwr wrote:
DavidG@VeritasPrep wrote: I'm not quite sure what you're asking here. Are you proposing a scenario in which the above appears as part of the original argument and wondering if such a premise would be ironclad?
Premise: Investors believe that the bank is going to fail
Conclusion: Therefore, there will be a run on the bank

Q: We are asked to Weaken the Conclusion.

One of the answer choice is: a recently aired news that the banks may get govt support raised doubts among investors.

Can I straight way eliminate such a choice on the basis that it raises doubts on the Validity of the premise?
If a premise says that the investors believe the bank is going to fail and an answer choice stipulates investors, in fact, have confidence in the bank, because the government has pledged support, then yes, that answer would have to be wrong. (But if the investors had no confidence, it could be true that the investors are wrong about the bank's prospects.)
Veritas Prep | GMAT Instructor

Veritas Prep Reviews
Save $100 off any live Veritas Prep GMAT Course